Aussageformen

Erste Frage Aufrufe: 1144     Aktiv: 30.10.2019 um 12:26

0

Ich verzweifel gerade bei folgender Aufgabe, vielleicht kann mir ja jemand helfen:

 

Für welche Werte a,b ∈ IR ist die Aussageform (ax)^2 ≥ b über der Grundmenge IR

- allgemeingültig?

-  nicht allgemeingültig, aber erfüllbar?

- unerfüllbar?

 

Muss man dann für a und b einfach beliebig gewählte Werte einsetzen oder was wird hier als Lösung erwartet? 

 

Vielen Dank schon mal. 

LG 

gefragt

Punkte: 10

 
Kommentar schreiben
1 Antwort
0

Hallo,

wir haben die Ungleichung 

$$ (ax)^2 \geq b $$

gegeben. Dabei sind \( a,b \in \mathbb{R} \) Parameter. Wir sollen diese nun so anpassen, dass diese Ungleichung für jedes \( x \in \mathbb{R} \) gültig wird (allgemeingültig).

Nun gucken wir uns die Ungleichung an. \( a \) und \( x \) werden quadriert. Das bedeutet aber auch, das die linke Seite der Ungleichung niemals negativ wird. Nun setzen wir doch beispielsweise mal \( a =1 \) und \( b= -2 \). Wir erhalten:

$$ x^2 \geq -2 $$

Diese Ungleichung gilt nun für alle \( x \in \mathbb{R} \). Diese Aussage formulieren wir nun etwas allgemeiner.

Da die linke Seite niemals negativ wird, ist diese Aussage allgemeingültig, für \( a \in \mathbb{R} \) und \( b \in \mathbb{R}_{\leq 0} \).

Versuch mal den Rest. Ich gucke gerne nochmal drüber.

Edit: Das Bild zu meinem dritten Kommentar:

Grüße Christian

Diese Antwort melden
geantwortet

Sonstiger Berufsstatus, Punkte: 29.81K

 

Vielen Dank.

Dann anbei mal mein Versuch zum 3. Spiegelstrich "unerfüllbar":
wir haben die Ungleichung
(ax)^2≥b
gegeben.
Dabei sind a,b ∈ R Parameter. Wir sollen diese nun so anpassen, dass diese Ungleichung für jedes x∈R unerfüllbar wird.
a und x ist ein Produkt, welches 0 wird, wenn einer der beiden Multiplikatoren 0 ist. Nun setzen wir für a=0 und b=5 ein.

Wir erhalten:
0≥5
Da die linke Seite somit immer 0 ergibt, ist die Aussage unerfüllbar für alle a ∈ R=0 und b ∈ R>=0.


  ─   Jacqueline 24.10.2019 um 10:30

Fast, es ist \( b \in \mathbb{R}_{>0} \). \( b \) muss also echt größer sein, denn sonst hätten wir
$$ 0 \geq 0 $$
Das ist sogar allgemeingültig.
Ansonsten stimmt es :)

Was ist dann mit Punkt 2?

Grüße Christian
  ─   christian_strack 24.10.2019 um 11:23

Oh ok, vielen Dank, das leuchtet ein.

Ja bei Punkt 2 stehe ich noch ein bisschen auf dem Schlauch. Das finde ich schwieriger als Punkt 1 und Punkt 3. Da brauche ich ja eine bestimmte Zahlenspanne oder?
  ─   Jacqueline 25.10.2019 um 09:59

Durch den ersten Fall, wissen wir bereits das \( b > 0 \) sein muss, denn sonst ist die Ungleichung für alle \( a \) allgemeingültig. Also setzen wir schon mal \( b > 0 \)
Gucken wir uns nochmal ein Beispiel an. Wir setzen \( a = 1 \) und \( b = 1 \)
$$ x^2 \geq 1 $$
Für welche \( x \in \mathbb{R} \) ist diese Ungleichung nun lösbar?

Grüße Christian
  ─   christian_strack 25.10.2019 um 10:46

Für alle x ∈ R ≥ 1 ist diese Ungleichung lösbar. Und je nachdem was wir dann für ein a haben ist die Ungleichung somit teilweise erfüllbar und teilweise nicht erfüllbar. Somit wäre die Aussage nicht allgemeingültig, aber für bestimmte a für x ∈ R ≥ 1 und b ∈ R > 0 erfüllbar.

Komme ich der Lösung näher?
  ─   Jacqueline 30.10.2019 um 08:34

Stell dir die Ungleichung einmal als Parabel vor. Dafür setzen wir zuersts \( a \neq 0 \).

$$ f(x) = a^2 x^2 - b $$

Da \( a^2 \) stehts positiv ist, ist unsere Parabel nach oben geöffnet.
Die Ungleichung

$$ a^2 x^2 - b \geq 0 $$

beschreibt dann alle Werte oberhalb der \( x\)-Achse, die aber noch auf der Parbel liegen.
Ich habe oben in meine Antwort noch ein Bild eingefügt. Das schraffierte erfüllt die Ungleichung nicht(!!).
hätten wir jetzt beispielsweise ein negatives \( b \), so wäre der \( y\)-Achsenabschnitt oberhalb der \(x\)-Achse und somit wäre die komplette Parabel oberhalb der \( x-Achse \). Somit sind alle Werte der Parabel größer gleich Null (dies ist unser erster Fall gewesen).

Jetzt haben wir aber ein positives \( b \), also \( b> 0 \). Jede Parabel der Form
$$ a^2 x^2 - b $$
hat damit einen Abschnitt oberhalb der \( x\)-Achse und unterhalb. Ist somit also erüllbar, aber nicht allgemein gültig.

Bei Ungleichungen kannst du eine solche Überlegung immer zu rate ziehen, wenn du die Funktion einfach plotten kannst.

Um es nun zusammenzufassen. Der zweite Fall tritt ein, wenn
$$ a \in \mathbb{R} \backslash \{0\} \land b > 0 $$
gilt.

Grüße Christian
  ─   christian_strack 30.10.2019 um 12:24

Kommentar schreiben